sequence is defined by multiplying the previous term

This topic has expert replies
Master | Next Rank: 500 Posts
Posts: 234
Joined: Tue May 31, 2016 1:40 am
Thanked: 3 times
The next number in a certain sequence is defined by multiplying the previous term by some positive constant k, where k ≠ 1. How many of the first nine terms in this sequence are greater than 1?

(1) The ninth term in this sequence is 81.

(2) The fifth term in this sequence is 1.

OAB

Please explain.

User avatar
GMAT Instructor
Posts: 15539
Joined: Tue May 25, 2010 12:04 pm
Location: New York, NY
Thanked: 13060 times
Followed by:1906 members
GMAT Score:790

by GMATGuruNY » Sat Oct 15, 2016 11:24 pm
Needgmat wrote:The next number in a certain sequence is defined by multiplying the previous term by some positive constant k, where k ≠ 1. How many of the first nine terms in this sequence are greater than 1?

(1) The ninth term in this sequence is 81.

(2) The fifth term in this sequence is 1.

OAB

Please explain.
Statement 1: a₉ = 81
Case 1: k=81
a₈ = a₉/k = 81/81 = 1.
a₇ = a₈/k = 1/81.
a₆, a₅, a₄, a₃, a₂, and a� will all be less than 1/81 and thus less than 1.
Here, the first 7 terms are all less than 1, for a total of 7 terms less than 1.

Case 2: k=9
a₈ = a₉/k = 81/9 = 9.
a₇ = a₈/k = 9/9 = 1.
a₆ = a₇/k = 1/9.
a₅, a₄, a₃, a₂, and a� will be less than 1/9 and thus less than 1.
Here, the first 6 terms are all less than 1, for a total of 6 terms less than 1.

Since the number of terms less than 1 can be different values, INSUFFICIENT.

Statement 2: aâ‚… = 1
Case 3: k=2
a₆ = ka₅ = 2*1 = 2
aâ‚„ = aâ‚…/k = 1/2.
a₇, a₈ and a₉ will all be greater than 2 and thus greater than 1.
a₃, a₂, and a� will all be less than 1/2 and thus less than 1.
Here, the first 4 terms are all less than 1, for a total of 4 terms less than 1.

Case 4: k=1/2
a₆ = ka₅ = (1/2)(1) = 1/2.
aâ‚„ = aâ‚…/k = 1/(1/2) = 2.
a₇, a₈ and a₉ will all be less than 1/2 and thus less than 1.
a₃, a₂, and a� will all be greater than 2 and thus greater than 1.
Here, the last 4 terms are all less than 1, for a total of 4 terms less than 1.

Cases 3 and 4 illustrate that -- whether k is a positive integer or a positive fraction -- there will be a total of 4 terms less than 1.
SUFFICIENT.

The correct answer is B.
Private tutor exclusively for the GMAT and GRE, with over 20 years of experience.
Followed here and elsewhere by over 1900 test-takers.
I have worked with students based in the US, Australia, Taiwan, China, Tajikistan, Kuwait, Saudi Arabia -- a long list of countries.
My students have been admitted to HBS, CBS, Tuck, Yale, Stern, Fuqua -- a long list of top programs.

As a tutor, I don't simply teach you how I would approach problems.
I unlock the best way for YOU to solve problems.

For more information, please email me (Mitch Hunt) at [email protected].
Student Review #1
Student Review #2
Student Review #3

GMAT/MBA Expert

User avatar
Elite Legendary Member
Posts: 10392
Joined: Sun Jun 23, 2013 6:38 pm
Location: Palo Alto, CA
Thanked: 2867 times
Followed by:511 members
GMAT Score:800

by [email protected] » Sun Oct 16, 2016 10:37 am
Hi Needgmat,

This DS question tells us that each term in a sequence is equal to the PREVIOUS term multiplied by a POSITIVE CONSTANT.

For example, the sequence 1, 2, 4, 8, 16 would fit this definition (another example would be 16, 8, 4, 2, 1, 1/2, etc.). We don't know any of the terms though and we don't know the constant (it could be either an integer, fraction or mixed number). We DO know that since we're multiplying by a positive constant that the sequence of numbers either "increases" or "decreases."

We're asked how many of the first 9 terms are greater than 1? It's interesting that the question asks how many are greater than 1.

Fact 1: the 9th term = 81

IF k = 3, then the terms (working backwards from the 9th term....) are:
81, 27, 9, 3, 1, 1/3, 1/9, 1/27, 1/81
Here, the number of terms greater than 1 = 4

IF k = 1/3, then the terms (working backwards from the 9th terms....) are:
81, 243, 729, then they get bigger and bigger.....
Here, the number of terms greater than 1 = 9
Fact 1 is INSUFFICIENT

Fact 2: the 5th term = 1

Since the sequence either increases or decreases, we'd have...
4 numbers less than 1, 1, 4 numbers greater than 1
or
4 numbers greater than 1, 1, 4 numbers less than 1

Regardless of which option, we end up with exactly 4 terms.
Fact 2 is SUFFICIENT

Final Answer: B

GMAT assassins aren't born, they're made,
Rich
Contact Rich at [email protected]
Image

GMAT/MBA Expert

User avatar
GMAT Instructor
Posts: 3008
Joined: Mon Aug 22, 2016 6:19 am
Location: Grand Central / New York
Thanked: 470 times
Followed by:34 members

by Jay@ManhattanReview » Mon Dec 19, 2016 6:36 am
Needgmat wrote:The next number in a certain sequence is defined by multiplying the previous term by some positive constant k, where k ≠ 1. How many of the first nine terms in this sequence are greater than 1?

(1) The ninth term in this sequence is 81.

(2) The fifth term in this sequence is 1.

OAB

Please explain.
S1: We are given that T9 = 81 => T8 = 81/k, T7 = 81/k^2, ... T1 = 81/k^8. But we do not the value of k. Insufficient!

S2: We are given that T5 = 1 => T6 = k, T7 = k^2, ... T10 = k^5, .... But we do not the value of k. However, it is not insufficient.

If k > 1, then terms T6, T7, T8, and T9 are greater than 1. The answer is four.

If k < 1, then terms T1, T2, T3, and T4 are greater than 1. The answer is still four. We do not need to know the value of k. Sufficient!

We cannot reach a conclusion applying the same logic for S1. If k < 1, we get to know that there would be nine terms greater than 1. However, if k > 1, we cannot reach to a term lying between T1 and T9, that is less than 1.

Hope this helps!

-Jay

_________________
Manhattan Review GMAT Prep

Locations: New York | Jakarta | Nanjing | Berlin | and many more...

Schedule your free consultation with an experienced GMAT Prep Advisor! Click here.

GMAT/MBA Expert

User avatar
GMAT Instructor
Posts: 1462
Joined: Thu Apr 09, 2015 9:34 am
Location: New York, NY
Thanked: 39 times
Followed by:22 members

by Jeff@TargetTestPrep » Wed Dec 21, 2016 9:14 am
Needgmat wrote:The next number in a certain sequence is defined by multiplying the previous term by some positive constant k, where k ≠ 1. How many of the first nine terms in this sequence are greater than 1?

(1) The ninth term in this sequence is 81.

(2) The fifth term in this sequence is 1.
We are given that the next number in a certain sequence is defined by multiplying the previous term by some positive constant k, where k ≠ 1, and we need to determine how many of the first nine terms in this sequence are greater than 1.

Statement One Alone:

The ninth term in this sequence is 81.

Without knowing any other terms in the sequence or the value of the positive constant k we are multiplying, we can't determine the number of terms in the sequence that are greater than 1. For example, if k = 3, then by going backward we have:

9th term = 81, 8th term = 27, 7th term = 9, 6th term = 3, 5th term = 1, 4th term = 1/3, and so on.

In this case, we have 4 terms that are greater than 1.

However, if k = 9, then by going backward again we have:

9th term = 81, 8th term = 9, 7th term = 1, 6th term = 1/9, and so on.

In this case we have only 2 terms that are greater than 1. Therefore, statement one alone is not sufficient. We can eliminate answer choices A and D.

Statement Two Alone:

The fifth term in this sequence is 1.

Since there are 9 total terms, we see that the 5th term is the middle term in our sequence. In other words there are 4 terms below the 5th term and 4 terms above the 5th term. Since we know that k is positive, it's either a positive proper fraction or a number greater than 1. Thus, regardless of whether k is a positive proper fraction or a number greater than 1, there will be 4 numbers above 1 and 4 numbers below 1.

For instance, if k = 1/2, then the 1st to the 4th terms, inclusive, are greater than 1, and if k = 2, then the 6th to the 9th terms, inclusive, are greater than 1. Either way, there are 4 terms greater than 1. Statement two alone is sufficient to answer the question.

Answer: B

Jeffrey Miller
Head of GMAT Instruction
[email protected]

Image

See why Target Test Prep is rated 5 out of 5 stars on BEAT the GMAT. Read our reviews